LSAT and Law School Admissions Forum

Get expert LSAT preparation and law school admissions advice from PowerScore Test Preparation.

 Administrator
PowerScore Staff
  • PowerScore Staff
  • Posts: 8916
  • Joined: Feb 02, 2011
|
#33370
Complete Question Explanation

Parallel Flaw. The correct answer choice is (A)

This seemingly simple stimulus about a forest fire contains a fairly unique flaw. Based on the fact that investigators have not proved that campers started the fire, and that they have not proved that lightning caused the fire, the author concludes that the investigators have not proved that the fire was started by either campers or lightning:

  • Premise: ..... Investigators have not proven that the fire was started by campers.

    Premise: ..... Investigators have not proven that the fire was started by lightning.

    Conclusion: ..... Thus, investigators have not proven that the fire was started by either campers or lightning.
The problem with this reasoning is that the author ignores the possibility that investigators could have narrowed down the possibilities to exactly two potential culprits: campers, or lightning. Thus, when the author jumps to the conclusion that investigators have not proved that the fire was started by either campers or lightning, that conclusion is not valid.

The question stem asks for the answer choice whose reasoning most closely parallels the flawed reasoning in the stimulus. The correct answer choice will likely feature a conclusion that similarly fails to recognize that options could be narrowed down to exactly two possibilities.

Answer choice (A): This is the correct answer choice. Kim has no reason to think that Sada will be victorious in the upcoming election, and Kim has no reason to think that Brown will win. Based on these two premises, the author of this answer choice jumps to the conclusion that Kim has no reason to believe that either one will win the election:

  • Premise: ..... Kim has no basis to believe Sada will win.

    Premise: ..... Kim has no reason to believe that Brown will win.

    Conclusion: ..... Thus, Kim has no reason to believe that either Sada or Brown will win.


As with the stimulus, the reasoning in this answer choice invalidly concludes that there is no way to narrow down the possibilities to exactly two (for example, what if Sadi and Brown are the only two candidates in the upcoming election? Even without knowing who will win, Kim would still have reason to believe that either Sadi or Brown would win, given the lack of other possible options).

Answer choice (B): This choice looks appealing at first, as the premises are somewhat similar, but the conclusion does not parallel the flawed conclusion drawn in the stimulus. In this case, the premises are as follows:

  • Premise: ..... There is no proof that the thief escaped through the vent in the ceiling.

    Premise: ..... There is also no proof that the thief escaped through the window.

    Conclusion: ..... Therefore, these two possibilities are equally likely.
This conclusion is questionable; just because there is no proof that the thief took either path, that does not mean that the two possibilities are equally likely. However, although this choice does feature a questionable conclusion, it is not the same type of flaw as that found in the stimulus. Had this choice concluded that there is no proof that the thief escaped through either the vent or the window, this answer would have perfectly paralleled the flawed reasoning found in the stimulus. Since it does not, however, it should be ruled out of contention to answer this Parallel Flaw question.

Answer choice (C): The flaw reflected in this answer choice has a very clear structure based entirely on majorities:

  • Premise: ..... Most students in the dorm major in engineering.

    Premise: ..... Most students in the dorm are from out of town.

    Conclusion: ..... Most engineering majors in the dorm are from out of town.
This conclusion is flawed: If there are 100 people in the dorm, for simple reference let’s call them 1 through 100. Given the premises, it is possible that people 1-51 are engineering majors, and 49-100 are from out of town. This leaves the possibility of only a few engineering majors who are out of town (49-51, specifically)

Although the reasoning presented in this choice is flawed, it is not analogous to the flaw reflected in the stimulus, so it cannot be the right answer to this Parallel Flaw question.

Answer choice (D): The flawed reasoning in this stimulus bases a flawed conclusion on two very weak premises:

  • Premise: ..... In some parts of the forest, camping is permitted.

    Premise: ..... In some parts of the forest, hunting is permitted.

    Conclusion: ..... Thus, in some parts of the forest, both hunting and camping is permitted.
“Some” is a vague term; it could be that a very small section of the northeast corner of the forest, for example, is reserved for hunting, while a very small portion of the southwest corner is reserved for camping.

Although the reasoning in this choice is flawed, it does not share the very specific reasoning flaw found in the stimulus, so it can be eliminated from contention in response to this Parallel Flaw question.

Answer choice (E): The flawed reasoning presented in this choice is as follows:

  • Premise: ..... The car could have been driven by Jones when the accident occurred.

    Premise: ..... The car could have been driven by Katsarakis when the accident occurred.

    Conclusion: ..... Thus, the car could have been driven by both drivers at the same time (!)
Clearly, the fact that the car could have been driven by either driver does not necessarily support the conclusion that the car could have somehow been driven by both drivers, but this flaw does not match that of the reasoning in the stimulus, so it should be ruled out of contention.
 Applesaid
  • Posts: 29
  • Joined: Oct 18, 2013
|
#12696
Hello!

This is again a parallel flaw reasoning. I can somehow see the stimulus is problematic but I don't know why A is the correct answer choice?


Celine
 BethRibet
PowerScore Staff
  • PowerScore Staff
  • Posts: 200
  • Joined: Oct 17, 2012
|
#12720
Hi Celine,

Thanks for writing!

With this stimulus, we know essentially, A is not proven, and B is not proven, so there is no proof that either A or B must be correct. The flaw here is that while you might not have proven that one of two things is definitively true, it could be true that you had narrowed down to two options, one of which must be correct.

For instance, the police might know that only two people had the opportunity to commit a particular murder. They might not be able to prove that the first suspect did it rather than the first, or that the second suspect did it rather than the first, but by ruling out every alternate person or possibility, they might indeed have proven that it was one of the two.

Once you understand the flaw you're looking for, it should be easier to narrow down to answer choice A, which has the same basic pattern. A (Sada will win) is not established, B (Brown will win) is not established, so there is no reason to know that either A or B (one of the two will win) will be true. However, suppose these two, Sada and Brown, are the only ones running in the election, and people are out voting. Then indeed, we have reason to know that one of the two will win the election, even if we don't know yet which it will be.

Hope this helps!

Beth
 avengingangel
  • Posts: 275
  • Joined: Jun 14, 2016
|
#30926
Hello! I took this practice test today and got this answer right, but circled it because although I saw the pattern of reasoning, I didn't see the flaw in it. The explanation provided above has really helped me, though. Thanks! I guess I still feel a bit uneasy about this tho because I wasn't really aware of this as a "flaw" before right now. Is there a category of flaws this one falls "under"? I realize not every single flaw can be categorized, but it sure would be helpful to get a sense. Additionally, can you provide any other questions in other PTs (or the Course book or Bible), where we are asked to identify this type of flaw (so I can see how it's worded in a correct answer choice)? I feel like this is (somewhat) the opposite of the "false dilemma" flaw... ?

Any explanation or examples would be so helpful. Thanks!
 Adam Tyson
PowerScore Staff
  • PowerScore Staff
  • Posts: 5153
  • Joined: Apr 14, 2011
|
#31143
If I had to lump this into a category (which we don't have to do - those categories are there to help, not to box you in, and as long as you understand the problem it doesn't matter whether or not you can slap a label on it), I suppose I would call it a part to whole flaw. Just because something is true of each part (I can't prove it was part A or part B that did it) does not prove that the same thing is true of the whole (maybe I can prove that something within the combination of A and B, the whole, must have done it).

Don't let the labels be your goal, but rather make them just another tool that you can use if you need to. It's helpful when trying to conceptualize something if you can put a name to it - that's a shortcut to true understanding. True understanding doesn't require that shortcut, though, so as long as you "get" what you're looking at, there's no need to find a name to pin on it. There are way too many types of flaws for us to name them all neatly and tidily, or to categorize them for easy study. Use that tool when you can, and use other tools when those tools don't do the job.

Continued good luck!
 avengingangel
  • Posts: 275
  • Joined: Jun 14, 2016
|
#31918
Thanks!
 VamosRafa19
  • Posts: 39
  • Joined: Nov 14, 2020
|
#81456
Hi,

I picked the correct answer here but the thing that threw me off is that stem said that the pattern of reasoning in the stimulus was flawed but I couldn't really figure out why that was the case. It seemed fairly sound to me.
User avatar
 Stephanie Oswalt
PowerScore Staff
  • PowerScore Staff
  • Posts: 811
  • Joined: Jan 11, 2016
|
#81486
Hi VamosRafa19,

Thanks for the post! I have moved your question to the thread discussing this topic. Please review the above explanation, and let us know if that helps! Thanks!
 VamosRafa19
  • Posts: 39
  • Joined: Nov 14, 2020
|
#81581
Thanks Stephanie, that makes sense. I could have sworn I looked for the existing post for this question!
User avatar
 amys45
  • Posts: 10
  • Joined: Dec 22, 2020
|
#82863
Hi everyone,

Thank you for the explanations above, it definitely helped me understand the flaw in this question better! I do still have a question about this stimulus after reading the explanation, however.

I'm confused on why this argument is not valid given that the conclusion says that investigators have not proved that the blaze was caused by campers or lightning, and the reasoning given is two facts that say exactly this (that the investigators haven't proved the fire was started by either campers or lightning). I understand that it could be the case that investigators have narrowed it down to these two potential causes, but how does that invalidate the fact that we know the investigators have still not proved the forest fire was started by either campers or lightning?

I think I'm confused because I know we're supposed to take what's in the stimulus as given, and this line of reasoning seems to contradict that.

I would appreciate any help to see where I'm going wrong here!

Thank you!

Get the most out of your LSAT Prep Plus subscription.

Analyze and track your performance with our Testing and Analytics Package.